LSAT and Law School Admissions Forum

Get expert LSAT preparation and law school admissions advice from PowerScore Test Preparation.

 Administrator
PowerScore Staff
  • PowerScore Staff
  • Posts: 8917
  • Joined: Feb 02, 2011
|
#40670
Complete Question Explanation
(The complete setup for this game can be found here: lsat/viewtopic.php?t=15480)

The correct answer choice is (C)

In this List question, the rules should be applied in the following order: first, second, and third.

Answer choice (A): This answer choice violates the second rule and is therefore incorrect.

Answer choice (B): This answer choice violates the first part of the third rule and is therefore incorrect.

Answer choice (C): This is the correct answer choice.

Answer choice (D): This answer choice violates the second part of the third rule and is therefore incorrect.

Answer choice (E): This answer choice violates the first rule and is therefore incorrect.

There are only three rules in this game, so each of the incorrect answers cannot violate a different rule (there must be some overlap). But, not unexpectedly, two of the incorrect answers violate different parts of the third rule, so every part of each rule is tested in this question.

Get the most out of your LSAT Prep Plus subscription.

Analyze and track your performance with our Testing and Analytics Package.